Question NB2

Réussis tes devoirs et examens dès maintenant avec Quizwiz!

Given the functions of the posterior pituitary, what would the effects of a lesion to this structure?

(ADH) —> impaired vasodilation and water reabsorption —> low BP, dehydration, low sodium (Oxytocin) —> Uterine relaxes during labor, no milk ejection during feeding, no bond with mother and newborn, anxiety, depression

A space occupying lesion in the nasopharyngeal could lead to chinch of the following problems? Answer with Y/N and explain -Blocking of nostril (lead to snoring and dry mouth) -Loss of smell/taste -Bad breath -Sleep apnea -Block Eustachian tube (lead to hearing and otitis media problems with effusion) -Nosebleed -Cranial nerve lesions

-Blocking of nostril (lead to snoring and dry mouth) —> YES -Loss of smell/taste (CN 7,9,10) —> NO -Bad breath —> NO -Sleep apnea —> YES -Block Eustachian tube (lead to hearing and otitis media problems with effusion) —> NO (won't rupture tympanic membrane) -Nosebleed —> YES -Cranial nerve lesions —> YES

In which layer of fascia are the muscles innervated by the spinal accessory nerve?

-investing fascia

What is the embryological origin of the hyoid bone?

2nd pharyngeal arch

A 4-year-old girl comes to the maxillofacial surgery department for the progressive repair of congenital facial and oral abnormalities that have caused breathing, feeding and speech difficulties in the past. The surgeon suspects that these abnormal facial features are because of the mandible's faulty development. Which of the following pharyngeal arches was most likely affected in this patient? A. 1st B. 2nd C. 3rd D. 4th E. 5th F. 6th

A. 1st

A 60-year old man comes to the physician because he notices small amounts of flood in his oral rinse after brushing his teeth. Oral exam reveals small cutes on the lateral surfaces, body and tip of the tongue. There are no bleeding gums or dental cavities. CN exam shows ability to taste, but he doesn't respond to general sensory input on the anterior 2/3 of the tongue. The nerve responsible for general sensation to affected region of the tongue is derived from which other following pharyngeal arches? A. 1st B. 2nd C. 3rd D. 4th E. 5th F. 6th

A. 1st

A newborn is being assessed by the physician immediately after birth. His face is pink, while the extremities are bluish in color. He is breathing spontaneously but irregular with occasional grunting. Vigorous stimulation only produces a weak grimace. His limbs are limp. Umbilical pulse is estimated at 85/min. What is the Apgar score for this newborn? A. 4 B. 5 C. 6 D. 7 E. 8

A. 4 -face is pink, extremities are blue (-1) -irregular breathing with occasional grunting -weak grimace -limp limbs -85/min

An 18-year-old man comes to the physician for a follow-up examination. He is participating in a research study on normal brain development and has undergone MRIs of the brain biannually since the age of 6 until now. Which of the following best characterize his brain development during this time frame? A. A linear increase in myelination B. An inverted U-shaped trajectory characterized initially by a decrease in grey matter followed by an increase C. A linear increase in grey matter D. A linear decrease in grey matter E. A steady increase in CSF volume

A. A linear increase in myelination

What would be the effect of a lesion at the following locations? A. Proximal to the branching of the pharyngeal branches of the vagus nerve B. Distal to the pharyngeal breaches but superior to the superior laryngeal nerve C. Distal to the superior laryngeal nerve

A. Affect digestion, hoarseness, difficulty swallowing (palatine muscles not intact) B. Difficulty speaking, hoarseness (palatine muscle intact above) C. Difficulty breathing, possible issue with digestion (voice cord not affected)

A 15-year-old girl is brought to the physician by her mother because of a 1-day history of headache, facial pain along her cheekbones and vision problems. She has an infected pimple on the ala of her nose which she repeatedly picks against her mother's advice. A diagnosis of cavernous sinus thrombosis is made. Which of the following veins is the most likely route that introduced the infection to the cranial cavity? A. Angular vein B. Superior sagittal sinus C. Lingual vein D. Superior petrosal sinus E. Internal jugular veins

A. Angular vein

A 1-year-old boy is brought to the physician by his mother because his head appears to be growing abnormally and she is concerned. An image of his head is shown. Which of the following is the most likely cause of the patient's condition? A. Bilateral premature fusion of the coronal suture B. Unilateral premature fusion of the coronal suture C. Premature fusion of the sagittal suture D. Premature fusion of the lambdoid suture E. Premature fusion of the frontal suture

A. Bilateral premature fusion of the coronal suture

A. Compare and contrast punishment and operant extinction B. Compare and contrast classical extinction and operant extinction

A. Both decrease a behavior, however operant extinction is when the behavior decreases because reinforcer stops, whereas punishment is an event/response/consequence d/t a behavior B. Both weaken/eliminate a behavior, but CC has 2 stimuli that are paired with a previous neutral stimulus (CS) whereas OC is d/t a consequence of a behavior

During a neurological exam, the physician tests for taste in the anterior 2/3rds of the tongue. The patient doesn't taste anything and complains to the physician that he has been experiencing dry mouth while eating. Which of the following best describes the nerve responsible for both of these symptoms? A. Chorda tympani nerve from facial that accompany the lingual nerve B. Greater petrosal nerve from facial that accompany glossopharyngeal C. Inferior alveolar nerve from the mandibular D. The maxillary division of the trigeminal nerve E. The info petrosal verve from the facial that accompany lingual

A. Chorda tympani nerve from facial that accompany the lingual nerve

An infant is delivered by emergency Caesarean section because of pre-eclampsia. Apgar examination at 5 minutes after delivery shows the infant's body is pink with blue extremities. Pulse is 102 per min. The infant responds promptly responds to painful stimulation with a grimace, has active muscle tone, and is crying loudly. The infant's rating on which of the following sections of the Apgar examination is most likely to reduce her score from a perfect 10? A. Color B. Heart rate C. Response to painful stimulation D. Muscle tone E. Respiration

A. Color

A 73-year-old woman is brought to the physician by her daughter for follow-up assessment after she fell and hit her head 3 days prior. At the end of the appointment, the daughter is frustrated with the physician because she believes the physician does not care about her mother's health. Given this frustration, which of the following is the best course of action by the physician? A. Delay the end of the appointment, acknowledge the daughter's feelings, and ask her to elaborate on why she believes the physician does not care about her mother B. Acknowledge her concerns and explain that there is nothing that can be done C. Acknowledge her concerns and explain that it was an exceptionally busy day in the practice D. Deny her claim and explain that her mother is receiving excellent care E. Promise her it will be better next time

A. Delay the end of the appointment, acknowledge the daughter's feelings, and ask her to elaborate on why she believes the physician does not care about her mother

A 50 y/o woman is brought to the physician by her husband because of 2 day hx of strange behavior. In response to a question about her sleep habits, she quickly says, "sleep is nice. I enjoy counting sheep. I have a lovely wool sweater." Which of the following terms best describes her thought process? A. Flight of ideas B. Obsessional C. Neologistic D. Paraphasic E. Clanging

A. Flight of ideas

A 37-year-old woman comes to the physician because of long-standing difficulties in conceiving. She reports a successful professional career but worries about her reproductive capabilities. She also reports envy that her younger sister is raising two healthy children. Her parents, who visit daily, make unpleasant comments regarding their desires for more grandchildren. Insufficient hypothalamic release of which of the following hormones is leads to the the inability to conceive? A. GnRH B. LH C. FSH D. Estrogen E. Progesterone

A. GnRH

A 12-year-old boy is brought to the emergency department because of head trauma after motor vehicle accident 1 hour ago. Temperature 37.6 C , pulse is 70/min, blood pressure is 128/78 mm Hg, respiration is 18/mi. Physical examination shows bruising around the mastoid process, raccoon eyes and Cerebrospinal fluid rhinorrhea. Subsequent physical examination reveals he is devoid of emotional tearing on eye. Which of the following nerves is most likely damaged? A. Greater petrosal nerve B. Lesser petrosal nerve C. Zygomatic nerve D. Lacrimal nerve E. Chorda tympani nerve

A. Greater petrosal nerve

A 43 year old man presents with loss of control of facial expression across the entire right side. The corner of his mouth droops on the right side, but he can clench his jaw and chew on demand. During examination, his ohysician also notes loss of hearing on the right side, and the patient has difficulty maintaining balance while standing on one foot. The patients corneal (blink) reflex is absent in the right eye, but cutaneous sensation is normal on the entire face. The physician orders radiograohic imaging in anticipation of finding a tumor. What is the most likely location of the tumor? A. Internal acoustic meatus B. Foramen ovale C. Foramen rotundum D. Geniculum of the facial canal E. Stylomastoid foramen

A. Internal acoustic meatus

Name the neurotransmitters and their receptors using similar processes during neurotransmission A. Ionotropic B. Metabotropic

A. Ionotropic (Salty/sour; ion channels with current through it) B. Metabotropic (Sweet/bitter; GPCRor IP3)

The mental nerve has which of the following characteristics? A. Is a branch of a nerve that passes through a foramen of the sphenoid bone B. Is a branch of CN VII C. Is having sensory and motor fibers D. Contains parasympathetic fibers E. Is a nerve that innervates muscles of facial expression

A. Is a branch of a nerve that passes through a foramen of the sphenoid bone

A 36-year-old woman is brought to the emergency department with severe head injuries after a car crash. Physical examination shows a deviated uvula to the right. Which nerve is most likely affected to result in this deviation? A. Left vagus B. Right vagus C. Right hypoglossal D. Left glossopharyngeal E. Right glossopharyngeal

A. Left vagus

Discuss the terms: A. Pharyngeal pouch B. Pharyngeal groove (cleft) C. Pharyngeal arch Review the clinical significance of these structures:

A. Look back at grooves and pouch derivative B. Look back at grooves and pouch derivative C. Look back at the arch derivatives

During a phase 2 clinical trial for a new antihypertensive drug, participants are placed initially on a once daily dosing regime and documentation is made of the side effect profile. Upon taking the drug, participants discover getting instant eye and head pains, loss of vision, redness of the eye and the appearance of rainbows ("halo") around light sources in comparison to the control group. Which of the following best characterizes the mechanism of this observed side effect? A. Obstruction of the flow of aqueous humor B. Opacification of the lens C. Degeneration of the macula D. Loss of blood supply to the retina E. Inflammation of the conjunctiva

A. Obstruction of the flow of aqueous humor

A 56 year old woman is brought to the emergency room by her son who found her semi-conscious in her room. PE showed right upper eyelid was unable to lift spontaneously. After lifting the lid, the eye was seen to be deviated laterally and downward with a dilated pupil. Which of the following structures was most likely affected to result in these symptoms? A. Oculomotor n. B. Optic n. C. Facial n. D. Ciliary ganglion E. Superior cervical ganglion

A. Oculomotor n. (Motor) -can't lift eyelid; levator pupillae superiorlis -deviated laterally and downward = oculomotor problem -After leaving the spinal cord, the fibres enter the sympathetic chain. This structure spans from the base of the skull to the coccyx, and is formed by nerve fibres and ganglia (collections of nerve cell bodies). There are three ganglia within this chain that are of interest - the superior, middle and inferior cervical ganglia. The sympathetic fibres synapse with these ganglia, with post ganglionic branches continuing into the head and neck (IF THE LESION WAS IN E, THEN THE PATIENT WOULD HAVE HORNERS SYNDROME) -none of the motor fibers of the cranial nerve are sympathetic. Only somatomotor or parasympathetic - LR (ABduction) & SO (depression) are unopposed by the muscles and aren't innervated by oculomotor n.

A. What is meant by craniosynostosis? B. What are the different types? C. What are the main features of each type?

A. One or more of the sutures fuses prematurely , changing the growth pattern of the skull B. Trigoncephaly (early closure of frontal suture) Scaphocephaly, Bradycephaly, Plagiocephaly, Cranium Bifidum (w/meningocoele, meninigoencephalocoele, or meningohydroenceohalocoele)

A 7-year-old boy is brought to the physician by his mother for a routine examination. His BMI is above the 95th percentile, so the physician recommends putting him on a diet. The mother has been giving him candy every day for years because he whines if he does not get any, but she pledges to stop giving him candy even if he whines. After about a week, he no longer whines about not getting candy. Which of the following learning principles best explains why his whining behavior diminished? A. Operant extinction B. Negative punishment C. Positive punishment D. Extinction burst E. Classical extinction

A. Operant extinction

A 6-year-old boy comes to the hospital to repair congenital facial and oral abnormalities that have caused respiratory, feeding and speech difficulties in the past. The pediatrician suspects that one of his facial features is because of the faulty development of the frontonasal process. Which of the following was most likely derived from this embryologic structure? A. Philtrum of the lip B. Lower lip C. Cheek D. Maxilla E. Secondary palate

A. Philtrum of the lip

A 30-year-old woman is brought to the emergency room because of a 1-day history of strange behavior. She has been yelling on a street corner for more than 24 hours straight. During examination, she angrily states that the only reason she is in the hospital is because society is "conspiring against her." She describes herself as "tuned in to a higher form of reality" which she claims frightens people because she is "the only one who God is communicating with." Which of the following statements is most likely the characterization of her level of insight? A. Poor, because she does not show any awareness of her psychiatric symptoms B. Good, because she has an acute awareness of society's problems. C. Moderate, because she understands that she has symptoms D. Good, because she knows that if she denies any symptoms she can be discharged

A. Poor, because she does not show any awareness of her psychiatric symptoms

Below are 5 consequences to a behavior. Label the consequence as either a reinforcer or a punishe and identify whether the reinforcer/punishe is positive or negative. A. Getting reprimanded because of tardiness B. Getting complimented because of weight loss C. Feeling less anxious about germs after washing ones hand D. Feeling excited by going to a sporting event E. Losing exam points because of unprofessional behavior

A. Positive punishment B. Positive reinforcement C. Negative reinforcement D. Positive reinforcement E. Negative punishment

A 72-year-old woman comes to the physician for a follow-up examination. She had a meningioma located near the left hypoglossal canal which was surgically removed, 2-weeks ago. Which of the following will most likely be tested on neurological examination? A. Resistance to a tongue spatula moving the tongue sideways B. The function of the thyrohyoid muscles C. Deviation of the uvula D. Ability to shrug her shoulders E. Taste on the anterior two third of the tongue F. Ability to turn her left ear towards her left shoulder

A. Resistance to a tongue spatula moving the tongue sideways

A 15-year-old boy is brought to the physician by his parents because of a 2-week history of being unable to rotate his head to the left side. He received a stab wound to his neck just posterior to the sternocleidomastoid during a fight at school 3 weeks ago. Which of the following nerves was most likely damaged by the knife? A. Right spinal accessory B. Left spinal accessory C. Right vagus D. Left vagus E. Ansa cervicalis

A. Right spinal accessory

Review the clinical significance of these structures: A. Pharyngeal piouch B. Pharyngeal groove (cleft) C. Pharyngeal arch

A. Separate the arches internally and lined by endoderm B. Separate the arches externally and lined by ectoderm C. Develop from mesoderm and migrating neural crest cells

A. How is skull development divided? B. Discuss the clinical significance of the fontanelles namely the anterior and posterior fontanelles.

A. Separated by sutures: frontal (metopic), coronal, sagittal and labmbdoid and develops into neurocranium (forms bones of cranial base and cranial vault) and viscerocranium ( forms bones of face) B. Anterior - The junction where the 2 frontal and parietal bones meet. The anterior fontanelle closes at 18-24 months. This fontanelle can be used to assess intracranial pressure due to e.g. intracranial masses . A sunken fontanelle is generally a sign of dehydration Posterior - The junction where the 2 parietal bones meet the occipital bone. This fontanelle generally closes before the anterior fontanelle at about 3 months.

A. What do we mean by socio-economic status? B. Why might this affect long term stress patterns?

A. Social standing or class of an individual or group. It is often measured as a combination of education, income and occupation B. If you don't have education or it's not as advanced as most people, then you have more stress because it can relate to having a lower income

A 3400-g female was born at 41 weeks' gestation to a primigravid mother. Labor was spontaneous and assisted with vacuum extraction because of prolonged second stage of labor. Apgars were 7 at 1 min and 9 at 5 min. Temperature 37.6 C , pulse is 120/min, blood pressure is 75/50 mm Hg, respiration is 34/min. 4 hours later the baby vitals was 37.6 C , pulse is 200/min, blood pressure is 55/35 mm Hg, respiration is 55/min. Physical examinations shows pallor with fluctuant scalp mass with increasing head circumference and periorbital ecchymosis. In which of the following areas is the fluid accumulating? A. Sub Galeal space B. Skin C. Galea aponeurotica D. Pericranium E. Subcutaneous layer

A. Sub Galeal space

A 3-year-old girl is brought to the physician by her mother because of a 3-month history of a swelling in the midline of her neck. Physical examination shows a midline mass that moves upwards when she is asked to protrude her tongue. Which of the following is the most likely diagnosis? A. Thyroglossal duct cyst B. Cervical (branchial) cyst C. Treacher Collin's syndrome D. Pierre Robin's syndrome

A. Thyroglossal duct cyst

A 22 y/o woman comes to the physicioan because of a 2 year hx of methamphetamine addiction. She usually uses the drug when she is at her boyfriends house. Now hen she merely sees her boyfriends house, she experiences physical and emotional changes that mimic the effects of methamphetamine. A. What is the conditioned response? B. Using classical conditioning technology, explain how to emilimate this conditioned response.

A. physical and emotional changes that mimic the effects of methamphetamine. US: methamphetamine addiction UR: expected symptoms CS: boyfriends house (used to be neutral) CR: physical and emotional changes mimicking the effects of meth B. Taking her to the house like it's a neutral stimulus so she doesn't have those negative effects by classical extinction

A 41-year-old man comes to the emergency department because of a 6-hour history of severe headache and nausea. He received a laceration while gardening 1 week ago, which he was self-treating. Physical examination shows a partially healed scalp laceration. Imaging studies show an infection of the superior sagittal sinus. Which of the following veins is most likely responsible for the patient's current infection? A.Emissary B.Inferior sagittal C.Diploic D.Basilar E.Intercavernous

A.Emissary

A 25-year-old man comes to the physician because of a 2-week history of numbness of the lower lip, left side of his face just anterior to his ear and difficulty chewing his food. The nerve responsible for innervation of the affected region most likely passes through which of the following foramina? A.Foramen ovale B.Foramen rotundum C.Internal acoustic meatus D.Stylomastoid foramen E.Mandibular foramen

A.Foramen ovale

Squeezing a pimple on the nose may lead to infection and disaster to follow. Explain the drainage and nerves and other structure that may become involved due to infection.

Angular vein drains to thalamic vein which drains to cavernous sinus—-> Leads to sensory loss

Review the development of the tongue

Anterior 2/3 of the tongue is from 1st arch (CN 5 - mandibular n.) with taste sensation coming for chorda tympani (CN 7) and general sense from CN 5 and posterior 1/3 of the tongue is from the 3rd arch (CN 9 - glossopharyngeal n.) with general/special sense from CN 9 and epiglottis from 4th arch (CN 10). 1st arch increases and goes over tuberculum impar while the 3rd arch outgrows the 2nd arch (copula). The muscles of the tongue create myoblasts from the occipital somites

A 16-year-old girl is brought to the physician by her mother because of a 2-day history of continuous severe pain in her right ear. She had an upper respiratory tract infection, 1 week ago. Otoscopic examination shows a red, bulging tympanic membrane on the right side. A diagnosis of otitis media is made. Which of the following is most likely the embryological origin of the structure that allowed the infection to reach the middle ear from the upper respiratory tract? A. 1st pharyngeal groove B. 1st pharyngeal pouch C. 2nd pharyngeal pouch D. 3rd pharyngeal pouch E. 4th pharyngeal pouch

B. 1st pharyngeal pouch

An infant is brought to the physician by his mother for a routine examination. Physical examination shows an infant who can hold his head upright and focuses on his hands. He smiles and coos with the physician. He is unable to pick up a cheerio with his thumb and index fingers. He does not crawl or sit unsupported. Which of the following is the most likely age of this infant? A. 1 month B. 4 months C. 6 months D. 9 months E. 12 months

B. 4 months

A newborn is being assessed by the physician immediately after birth. His face is pink, while the extremities are bluish in color. He is breathing spontaneously but irregular with occasional grunting. Vigorous stimulation only produces a weak grimace. His limbs are limp. Umbilical pulse is estimated at 140/min. What is the Apgar score for this newborn? A. 6 B. 7 C. 8 D. 9 E. 10

B. 7 Weak grimace = 1 Muscle tone = 2 Respiration = 1 Skin tone = 1 Pulse = 2

A 2-month-old boy is admitted to the hospital for investigation of a craniofacial anomaly. Physical examination shows a small mandible, cleft lip and palate and malformed external ears. He does not respond to auditory cues. A diagnosis of first arch syndrome is made. Which of the following would most likely be found on a CT scan, that explains his deafness? A. Malformed internal ear B. Absence of the malleus C. Large mandible D. Large tongue

B. Absence of the malleus

A 48-year-old woman comes to the physician because of a 4-day history of a lopsided face. Physical examination shows inability to close the left eye and a drooping of the left side of her mouth. Imaging studies show a mass located between the styloid and mastoid processes. Which of the following additional symptoms is most likely expected in this patient? A.Inability to chew on the left B.Change of depth of left nasolabial fold C.Difficulty swallowing D.Loss of sensation of the left face E.Impaired taste on the left anterior 2/3rds of the tongue

B. Change of depth of left nasolabial fold

A 12-year-old girl is brought to the physician by her mother because of a 2-month history of a mass in her neck. Her mother says that it was originally small, and peanut sized but the mass has grown painlessly during the past 2 months. Physical examination shows a smooth round mass the size of a golf ball located just anterior to the sternocleidomastoid. Which of the following is most likely the embryological basis for this condition? A. Persistence of the thyroglossal duct B. Failure of obliteration of part of the cervical sinus C. Failure of development of the second pharyngeal pouch D. Failure of development of the first arch E. Failure of development of the 3rd and 4th pharyngeal pouches

B. Failure of obliteration of part of the cervical sinus

A 4-year-old boy comes to the hospital to repair congenital facial and oral abnormalities that have caused several episodes of breathing and feeding difficulties. Physical examination shows a unilateral cleft in the posterior palate. Which of the following most likely occurred during this boy's embryologic development? A. The fusion of the lateral palatine processes with each other and the nasal septum B. Failure of the lateral palatine process to fuse with each other and the nasal septum C. Failure of the lateral palatine process to fuse with the primary palate, each other, and the nasal septum D. Failure of the maxillary prominences to fuse with the fused medial nasal prominences E. Failure of the maxillary prominence to fuse with the lateral nasal prominence

B. Failure of the lateral palatine process to fuse with each other and the nasal septum

A 23-year-old man is brought to the hospital by his brother because throat pain, fever and drooling of saliva for the last 2 hours. He had cough and fever for the last 2 days. Temperature 39.8 C (103.6F), pulse is 87/min, blood pressure is 145/88 mm Hg, respiration is 24/min. Physical exanimation shows edema, redness and tenderness of the palatine tonsils with cervical lymphadenopathy. Laboratory tests (rapid antigen test RADT) is positive and confirmed the diagnosis of group A Streptococcus bacteria. Which of the following never medicate the pain in this patient? A. Glossopharyngeal B. Glossopharyngeal nerve via pharyngeal plexuses C. Vagus nerve via pharyngeal plexuses D. Vagus nerve E. Pharyngeal branch of the maxillary nerve F. Recurrent laryngeal nerve G. Superior laryngeal nerve

B. Glossopharyngeal nerve via pharyngeal plexuses

A 46 year old woman with a brain lesion affecting the trigeminal nerve comes to the physician for evaluation. During the PE she doesn't respond to a piece of cotton brushed against her cheek and lower jaw. The physician approaches her eye with a tissue from the side to test the corneal reflex. When the tissue touches the corners, the patient doesn't respond. Which of the following is the correct explanation for the lack of response? A. The branch of the ophthalmic nerve innervating the eye is intact B. He branch of the ophthalmic nerve innervating the eye is damaged C. The branch of the trigeminal nerve responsible for innervating the orbicularis oculi is damaged D. The branch of the facial nerve responsible for innervating the orbicularis oculi is intact E. Nothing is supposed to happen, everything is normal

B. He branch of the ophthalmic nerve innervating the eye is damaged -facial nerve innervates motor movement of orbicularis oculi

A 54 year old man attends the clinic after experiencing chest pains and shortness of breath during a heated argument. Which of the following pathways is the most likely link between the argument and the chest pains? A. Nigrostriatal tract B. Hypothalamic-pituitary-adrenal (HPA) axis C. Solitary tract D. Hypothalamic-pituitary-gonadal (HPG) axis E. Spinocerebellar tract

B. Hypothalamic-pituitary-adrenal (HPA) axis

A 45-year-old women comes to the physician because of difficulty in swallowing, rapid breathing and change of voice (monotonous voice tone) for the last 2 weeks. She have a family history of multiple endocrine neoplasia type 2 (MEN II). Temperature 37.8 C (100.1F), pulse is 78/min, blood pressure is 145/88 mm Hg, respiration is 24/min. Physical examination shows a solitary thyroid nodule, 3x5 inches, extending retrosternally. Fine-needle aspiration (FNA) biopsy confirmed the diagnosis of medullary cell carcinoma. Which of the following blood vessels is related to affected nerve in this patient? A. Superior thyroid artery B. Inferior thyroid artery C. Right subclavian artery D. Ascending pharyngeal artery E. Thyroid ima artery F. Inferior laryngeal artery G. Internal jugular vein H. Right brachiocephalic veins

B. Inferior thyroid artery

A 58-year-old man comes to the physician because of a 6-month history of jaundice, fatigue, and weight loss and a 1-month history of abdominal pain. MRI of the abdomen shows stage I pancreatic cancer. The physician discusses the diagnosis and course of treatment with the patient, who responds that he is not interested in the recommended treatment but will instead use natural herbs and traditional Sikh treatments to cure his cancer. Which of the following is the best response to improve effective communication between the patient and physician? A. Change the treatment regimen to the herbs and traditional treatments the man prefers B. Inquire about the herbs and traditional treatments he plans on using and discuss options for combining them with your recommended treatment regimen C. Demand that avoid all herbs and traditional treatments and adhere to the physician-recommended treatment D. Refer him to a specialist in complimentary medicine. E. Set up a meeting between the man and a stage IV pancreatic cancer patient so he can see the gravity of his situation

B. Inquire about the herbs and traditional treatments he plans on using and discuss options for combining them with your recommended treatment regimen

A 21-year-old man is brought to the emergency department because of a 1-hour history of severe pain in his throat while breathing following the swallowing of extremely hot coffee which 'went the wrong way' and made him cough violently. Fiberoptic examination shows second degree burns in the vestibule of the larynx. Which of the following nerves most likely provides sensory innervation to this area? A. Pharyngeal plexus B. Internal laryngeal C. External laryngeal D. Recurrent laryngeal E. Glossopharyngeal

B. Internal laryngeal

The buccal nerve has which of the following characteristics? A. Is a branch of a nerve that passes through a foramen of the temporal bone B. Is a branch of CN Viii C. Is mainly motor with a small sensory component D. Contains parasympathetic fibers E. Is a nerve that innervates muscles of facial expression

B. Is a branch of CN Viii

A 47 year old man comes to the physician because of constant coughing, difficulty swallowing and occasional choking. PE shows deviation of the uvula when the patient says "ahh". Which of the following statements correctly describe the nerve that is damaged? A. It supplies the parasympathetic fibers to the parotid gland B. It serves as the efferent limb of the gag reflex C. It provides sensory innervation to the lower jaw D. It provides general sensory to the posterior q/e of the tongue E. It provides special sensory for taste to the anterior w/e of the tongue

B. It serves as the efferent limb of the gag reflex

A 37-year-old woman comes to the physician because of a 2-day history of double-vision. The double-vision is worst when she gazes to the right. The H-test shows weak abduction of the right eye. Which of the following extraocular muscles is most likely weak? A. Medial rectus B. Lateral rectus C. Superior rectus D. Inferior rectus E. Superior oblique F. Inferior oblique

B. Lateral rectus -think of what muscle is involved in ABduction of the right eye; also consider innervation, blood supply, sinuses and clinical correlates related to the muscle

A 33 y/o woman is brought to the ED by her husband b/c she threatened to "blow her brains out" 2 hours after losing her job. Her current alcohol level is 0.15. She is confused, repetitive and ataxic. She has a hx of self-cutting and prior suicidal ideation with no attempts. After spending the night in the hospital and detoxifying, she denies further suicidal intent. She doesn't own a gun or have access to one and she doesn't have a suicidal plan. Which of the following risk assessment best characterizes the suicide risk level in this patient? A. Low risk B. Moderate risk C. High risk

B. Moderate risk

A 45-year-old man comes to the physician for a follow-up examination. He has a 2-year history of diabetes, which he has been controlling through uncomfortable and inconvenient insulin injections. He has strictly adhered to his diet, and consequently he no longer needs to take insulin injections to manage his diabetes. Which of the following learning principles best explains his continued adherence to his diet? A. Positive reinforcement B. Negative reinforcement C. Positive punishment D. Negative punishment

B. Negative reinforcement

A 22-year-old man comes to the physician because of a 1-year history of posttraumatic stress disorder. His symptoms started 3 months after being released from a prison of war (POW) camp. Part of his symptomatology includes experiencing fear whenever he encounters anything that reminds him of his POW experience. He now persistently avoids such reminders to reduce his fear. Which of the following learning principles best explains his persistent behavior of avoiding reminders of his POW experience? A. Spontaneous recovery B. Operant conditioning C. Operant extinction D. Classical conditioning E. Classical extinction

B. Operant conditioning

A 22-year-old woman comes to the physician because of a 1-year history of anxiety. She is fearful of doing presentations in front of her class. In consultation with the patient the physician explains several different treatment options, including systematic desensitization, flooding, and exposure. Which of the following learning principles do all three of these approaches most likely involve? A. Gradually exposing the patient to a fear producing stimulus B. Presenting patient with a fear producing stimulus (simulated or in vivo) C. Training patient in a response incompatible with fear D. Positive punishment E. Negative reinforcement

B. Presenting patient with a fear producing stimulus (simulated or in vivo)

A 9-year-old girl is brought to the emergency department because of high fever and chest pain for the last 3 hours. She underwent palatine tonsils surgery removal 3 days ago. Temperature 39.2 C , pulse is 105/min, blood pressure is 135/84 mm Hg, respiration is 24/min. Physical examination shows tenderness on the anterior aspect of the neck, fever, stridor, drooling, enlarged cervical lymph nodes. Images are shown below. What is the most likely route for this infection to descend through the neck to reach the superior mediastinum? A. Parapharyngeal space B. Retropharyngeal space C. Buccal space D. Carotid sheath E. Suprasternal space

B. Retropharyngeal space

A 39-year-old man comes to the emergency department because of a sudden onset of nausea and some kind of "fuzzy vision". During primary gaze, he shows pathological nystagmus to the left, which gets worse during attempted gaze to the left. Neurological examination shows loss of pain and temperature on the left body and the right face. Imaging shows a lesion in the dorsolateral medulla. Which additional deficit is most likely present in this patient? A. Left anosmia B. Right anosmia C. Left ageusia D. Right ageusia

B. Right ageusia

A 25-year-old woman is brought to the emergency department because of motor vehicle accident 2 hours ago. Temperature 37.2 C , pulse is 85/min, blood pressure is 126/79 mm Hg, respiration is 19/min. Physical examination shows dropping and dry mouth. Imaging is shown below.Which of the following embryonic structures gives rise to the affected nerve? A. First pharyngeal arch B. Second pharyngeal arch C. Third pharyngeal arch D. Fourth pharyngeal arch E. Fifth pharyngeal arch

B. Second pharyngeal arch

A 17-year-old girl is brought to the physician by her mother because of a 3-week history of continuous nose bleeds. Physical examination shows a hyperemic area on the left anterior nasal septum, an area of anastomosis of blood vessels where the bleeding usually originates. Which of the following best describes the anastomotic vessels of the affected area? A. Sphenopalatine and anterior labial B. Sphenopalatine and superior labial C. Anterior ethmoidal and anterior labial D. Superior and anterior labial E. Sphenopalatine and middle meningeal

B. Sphenopalatine and superior labial

****A 58-year-old man comes to the physician because of a 6-hour history of loss of touch sensations in the right face, oral cavity and tongue. Imaging studies show a vascular lesion of the ventral posteromedial nucleus on the left. Which of the following additional sensory abnormalities is most likely present in this patient? A. Smell B. Taste C. Vision D. Hearing E. Balance

B. Taste

A young child suffered a debilitating o condition that includes progressive degeneration of the motor axons that innervated the masseter muscle. Which of the following muscles is most likely y to exhibit the same fate? A. Genioglossus B. Tensor tympani C. Orbicularis oris D. Levator veli palatini E. Stylopharyngeus

B. Tensor tympani -all come from the 1st arch -A = HYPOGLOSSAL -C = FACIAL N. -D = VAGUS -E = GLOSSOPHARYNGEAL

A 42 y/o woman comes to the physician because of a 3 month hx of anxiety. Her anxiety stated after being attacked by a stray dog while walking on the beach. She is now anxious whenever she is near the beach. Which of the following is most likely the conditioned stimulus? A. The dog attack B. The beach C. Walking D. Anxiety E. The stray dog

B. The beach

A midline sagittal CT image of the head and neck will show the health care worker which of the following consistent landmarks? A. The arythenoid cartilage articulating with the cricoid cartilage B. The body of the hyoid bone C. The hard palate at cervical vertebra level C6 D. Maxillary sinus located below the orbit E. Foramen ovale

B. The body of the hyoid bone

A 67-year-old man comes to the physician because of painful forehead rash and severe ocular pain for the last 24hr. Temperature 37.9 C , pulse is 75/min, blood pressure is 138/87 mm Hg, respiration is 19/min. Physical examination shows marked eyelid edema; conjunctival, episcleral, and circumcorneal conjunctival hyperemia and photophobia. What sensory ganglion is most likely affected in this patient? A. Otic ganglion B. Trigeminal ganglion C. Superior cervical ganglion D. Ciliary ganglion E. Submandibular ganglion

B. Trigeminal ganglion

A 48-year-old woman comes to the physician because of a 5-year history of anxiety and is treated with psychoanalytic therapy. Which of the following aspects will therapy sessions most likely focus on? A. Irrational thoughts and beliefs B. Unconscious drives and defense mechanisms C. Thoughts, beliefs and interpretation of events D. Positive reinforcement opportunities in life E. Negative self-schema

B. Unconscious drives and defense mechanisms

Acute stress induces energy mobilization. What might be the long-term metabolic effects of chronic stress? A. no effect as homeostasis adjusts to chronic stress B. increased blood glucose and fatty acids C. decreased blood glucose and fatty acids D. increased energy expenditure

B. increased blood glucose and fatty acids

A 23-year-old man has an infected gum opposite molar 2 on the left side of the lower jaw for two weeks. To which of the following structures will the lymph from this area initially drain to? A. submental lymph node B. jugulodigastric node C. supraclavicular nodes D. thoracic duct E. jugular trunk

B. jugulodigastric node

A 60-year-old man is brought to the emergency department by his grandson because of a 1-hour history of choking on a piece of steak during his dinner. His grandson performed the Heimlich maneuver without success and instead created an emergency laryngotomy with a steak knife. If this emergency airway is created in the cricothyroid membrane, in which of the following locations would it most likely be immediately located? A.Superior to the thyroid cartilage. B.Inferior to the thyroid cartilage. C.Inferior to the cricoid cartilage. D.Superior to the thyroid isthmus. E.Inferior to the thyroid isthmus.

B.Inferior to the thyroid cartilage.

A 12-year-old boy is brought to the physician by his mother because of a 2-day history of severe headaches and a feeling of heaviness in the mid-part of his face. He has a history of a recent bacterial upper respiratory tract infection. Imaging studies show fluid accumulation in the paranasal sinuses. Which of the following statements best describes the normal drainage of the paranasal sinuses? A.Maxillary sinus drains into the inferior meatus B.Maxillary sinus drains into the middle meatus through the semilunar hiatus C.Frontal sinus drains into the superior meatus D.Anterior and middle ethmoid also air cells drain into the spheno-ethmoidal recess E.Frontal sinus drains into the inferior meatus through the semilunar hiatus

B.Maxillary sinus drains into the middle meatus through the semilunar hiatus

Why is it difficult to break the lamina papyracea and not so much the floor of the orbit?

Blow-out fractures occur when a blow to the eye increases pressure in the orbit, causing the weak floor or medial wall (lamina papyracea) to "blow out" into the maxillary sinus or ethmoid bone. This results in a fracture, though it often prevents globe rupture and loss of the eye. Basically, a fracture here could entrap he medial rectus. Fracture to the floor of the orbit would entrap the inferior rectus muscle.

What is the difference between buccal nerve and the buccal branch?

Branch = CN 7 Nerve = CN 5

A 25-year-old woman comes to the physician because of a 2-week history of panic attacks. She has been experiencing sudden onset of shortness of breath, increased heart rate and chest pains. Which of the following is the most appropriate initial response from the physician? A. "Are you under a lot of stress at work?" B. "I understand that these experiences might be scary for you. Are you sleeping enough?" C. "Tell me more about your panic attacks." D. "I think you should try meditation and yoga." E. "Are you having any relationship problems?"

C. "Tell me more about your panic attacks."

A 47 year old wan is brought to the ED because of problems with speech and pain of her lower jaw for the last 2 days. She has a history of MS for the last 10 years. She stated that the pain is triggered by eating, talking or brushing her teeth and offer gets worse as the day progresses, which nerve is the source of their pain? A. 1st division of trigeminal n. B. 2nd division of trigeminal n. C. 3rd division of trigeminal n. D. Buccal branch of facial n. E. Marginal Mandibular branch of facial n.

C. 3rd division of trigeminal n. Inferior alveolar n. innervating teeth

Which group do you think experienced the greatest reduction in low back pain following treatment? A. Active IFC, Limited Therapeutic Alliance (AL group) B. Sham IFC, Limited Therapeutic Alliance (SL group) C. Active IFC, Enhanced Therapeutic Alliance (AE group) D. Sham IFC, Enhanced Therapeutic Alliance (SE group)

C. Active IFC, Enhanced Therapeutic Alliance (AE group)

A 61 year old male with terminal cancer comes to the physician because he wants to end his life before the physical and emotional pain from his illness becomes any more difficult to bear. He describes having difficulty getting out of bed in the morning, lack of interest in activities he used to enjoy, feeling of worthlessness, insomnia, poor concentration, and substantial regrets not having lived a more meaningful life. Which of the following states of grief most likely preceded his present state? A. Denial B. Anger C. Bargaining D. Depression E. Acceptance

C. Bargaining

In a typical cranial nerve sensory neuron chain in which of the following locations of the first and second order neuron synapse with each other? A. Cranial nerve ganglion B. Parasympathetic ganglia C. Brainstem nucleus D. Thalamus

C. Brainstem nucleus

A 45-year-old woman comes to the physician for a follow-up examination. She has been receiving chemotherapy for breast cancer for the past 2 months. After each chemotherapy session, she feels nauseous. Now, the sights, smells, and sounds of the treatment room make her feel nauseous even before the chemotherapy session begins. Which of the following is most likely serving as the unconditioned stimulus? A. Sights, smells, and sounds of the treatment room B. Nausea C. Chemotherapy D. Breast cancer E. Follow-up examination

C. Chemotherapy

A 68 year old man was choking on a piece of a steak at a family restaurant. Despite attempts to dislodge the foood via abdominal thrust (Heimlich maneuver), his upper airway remained block. An EMT, eating at the scene, performed an emergency tracheotomy enable the man to breathe. Which subcutaneous structure was must likely during this procedure? A. Cricoid cartilage B. Thyrohyoid membrane C. Cricothyroid membrane D. Tracheal rings E. Isthmus of thyroid gland

C. Cricothyroid membrane (cricothyroidotomy —> type of tracheotomy) -A = cartilage is very tough and avascular, so it would take too long to cut -B = superior laryngeal a. and internal h of the same artery innervates that membrane, also it's too high up from where the blockage is -C = b/w cricoid and thyroid, which is the best place - D = tracheal ring; tracheostomy = sterile conditions with surgical opening with a tube inserted. NEED ANESTHESIA because you're creating a stoma -E = too many complications because it's an endocrine organ, which can cause more problems in a non-sterile area

A 72 year old woman comes to the physician because of a 12 month history of memory problems. She has difficulty finding the right word in conversations and remembering the names of new people that she meets. Mental status exam shows that she is experiencing normal age-related cognitive decline. Which of the following structural brain changes is most likely to be associated with her normal age-related cognitive decline? A. Synaptic pruning B. Apoptosis C. Demyelination D. Neurogenesis E. Decreased CSF

C. Demyelination

A 55-year-old man comes to the physician because of a 2-day history of blood-shot eyes, headache and generalized facial pain. He has a recurrent nasal abscess which he continually ruptures to drain fluid. A diagnosis of cavernous sinus thrombosis is made. Which of the following additional symptoms would most likely be found on physical examination? A. Decreased tone and strength of muscles of mastication B. Decreased sensation over the lower jaw C. Diplopia D. Decreased hearing in both ears E. Deviation of the uvula when the patient is asked to say 'ahhh'

C. Diplopia

Which of the following is an example of a somatic motor cranial nerve nucleus? A. Edinger-Westphal B. Dorsal nucleus of vagus C. Facial nucleus D. Solitary nucleus E. Spinal trigeminal

C. Facial nucleus

A 45 year old woman comes to the physician because of a 6 month history of depressed mood and suicidal ideation. She is divorced and without children. She was not able to travel the works like she had planned. She is most likely functioning in which of the following psychosocial stages? A. Integrity vs. despair B. Identity vs. role confusion C. Generativity vs.. Stagnation D. Competence vs. inferiority E. Initiative vs. guilt F. Intimacy vs. isolation

C. Generativity vs.. Stagnation

A 27 y/o woman comes to the physician because of a 1 month hx of emotional upset and anxiety. Her boyfriend broke up with her after they had been together for 5 years, she is convinced that she will never be in another relationship because no one will want her. She says, "I will be alone forever." Based on cognitive therapy, which of the following is most likely the source of the woman's distress A. Her boyfriend breaking up with her B. Her lack of emotional resilience C. Her irrational belief that this event is catastrophic D. The unlikelihood of the woman being in another relationship

C. Her irrational belief that this event is catastrophic

A 55-year-old woman comes to the physician for a routine examination. She has not developed a stable sense of self. Since adolescence, her values, opinions, and career choices are constantly changing. Which of the following stages of psychosocial development is most likely relevant to the formation of her dominant personality traits? A. Autonomy vs shame and doubt B. Industry vs inferiority C. Identity vs role confusion D. Intimacy vs isolation E. Generativity vs stagnation F. Integrity vs despair

C. Identity vs role confusion

A 31-year-old woman comes to the clinic because of 30-day history of restlessness and frequent anxiety. She does not sleep well throughout the night and is tired throughout most of the day. She is also concerned about being unable to conceive during the last year. Her blood pressure is 140/110 mmHg and pulse 94/min. She is diagnosed with chronic stress and is asked to adopt some lifestyle changes. Which of the following changes best represents a long-term physiologic effect due to her chronic stress? A. Decreased plasma glucose and increased plasma free fatty acids B. Increased plasma glucose and decreased plasma free fatty acids C. Increased plasma glucose and increased plasma free fatty acids D. Decreased oxidation of glucose and free fatty acids E. Decreased insulin resistance

C. Increased plasma glucose and increased plasma free fatty acids

A 21 year old man comes tot he physician because of a shooting pain in his lower jar and complains it's the most severe pain he has a ever felt without muscle weakness. PE shouts no visible lesions or loss of strength/movement of muscle of mastication. Detailed patient Hx shows that the pain is sharp shooting pain localized on the right side from his ear along the side of his chin. The patient explains that the pain is intermittent but lasts for several days at a time. A diagnostic of trigeminal neuralgia is made. Which of the following statements regarding the affected nerve is correct? A. It exits the skull though the foramen rotundum B. It exits the brainstem at the pontomedullary junction C. It's the third branch of the ganglion D. It contains only motor fibers

C. It's the third branch of the ganglion

A 47-year-old man comes to the emergency department because of a 3-week history of headaches. Neurological examination shows abnormalities of ocular reflexes. When the left cornea is touched with a cotton wisp, only the right eye blinks. When the right cornea is touched, only the right eye blinks. Neutral gaze is normal. Which of the following structures is most likely lesioned? A. Left trigeminal ganglion B. Right trigeminal ganglion C. Left internal acoustic meatus D. Right internal acoustic meatus E. Left facial colliculus F. Right facial colliculus

C. Left internal acoustic meatus - injury to facial colluiculus would cause the dame isssue but lateral deviation and this case has neutral gaze, it's not impaired

A 22-year-old woman is brought to the emergency department by ambulance because of a 1-hour history of diabetic coma. When stabilized, she says that she had not been adhering to her physician's diet and exercise recommendations for her diabetes. She does not like her physician because the physician reminds her of her mother. She has not spoken to her mother since she left home at the age of 16 due to constant disagreements. Which of the following psychological factors most likely contributed to the patient's poor treatment adherence? A. Negative countertransference B. Positive countertransference C. Negative transference D. Positive transference E. Identification

C. Negative transference

A 25-year-old man comes to the physician because of a 6-month history of worsening anxiety. He has a 3-year history of general anxiety disorder and has been in partial remission until recently. He is tachycardic and diaphoretic. Which of the following components of his presentation are most likely his objective and subjective findings, respectively? A. Objective: Worsening anxiety Subjective: Tachycardic, diaphoretic B. Objective: Tachycardic, diaphoretic, worsening anxiety Subjective: Generalized anxiety disorder C. Objective: Tachycardic, diaphoretic Subjective: Worsening anxiety D. Objective: Generalized anxiety disorder Subjective: Tachycardic, diaphoretic, worsening anxiety E. Objective: Worsening anxiety, tachycardic Subjective: Diaphoretic F. Objective: Diaphoretic Subjective: Worsening anxiety, tachycardic

C. Objective: Tachycardic, diaphoretic Subjective: Worsening anxiety

A 10-year-old boy is brought to the physician by his parents because of a 2-day history of fever and intense pain around his ear. Physical examination shows a large swelling antero-inferior to the ear which is tender and warm to the touch. He is unable to tightly close his left eye and cannot frown his forehead on the same side. Which of the following structures is most likely infected resulting in the patient's symptoms? A. Temporalis muscle B. Facial vein C. Parotid gland D. Submandibular gland E. Masseter muscle

C. Parotid gland

A 43-year-old man comes to the physician for a routine exam. His BMI is 27 kg/m2. He leads a sedentary lifestyle and seldomly exercises. The physician points out the potential benefits of a healthy lifestyle and asks whether he has considered becoming more physically active. He has tried many times to become more physically fit but has always become discouraged and given up. He is now in a new relationship and wants to be in better physical shape. He discusses with the physician possible weight loss options and agrees upon a date to begin his exercise regimen. Which of the following stages of behavior change is he most likely in? A. Precontemplation B. Contemplation C. Preparation D. Action E. Maintenance

C. Preparation

A 37-year-old women comes to the physician for a routine check-up. She learns that, after 40 years of practice in a small Indiana town (population: 4,000), her physician plans to retire. Which of the following is the best course of action to ensure the physician does not abandon her patients? A. Throw a farewell party the week before she retires B. Send a letter to each of her patients to week before she retires C. Send a registered letter to each of her patients 3 months before she retires D. Post a sign on her practice door once she retires E. Do nothing

C. Send a registered letter to each of her patients 3 months before she retires

A 67-year-old man comes to the physician because of a 6-month history of progressive hearing loss. He has been a night club worker for the past 43 years. Physical examination shows lateralization of the Weber's test to the right side. Rinne's test was also positive in the right ear. MRI shows an acoustic neuroma associated with the structure shown in the attached image. Which of the following structures is most likely affected? A. Semicircular canal B. Crista ampullaris C. Spiral ganglion D. Macula of the utricle E. Macula of the saccule

C. Spiral ganglion

A 45 y/o woman comes to the physician because of a 1 month hx of anxiety. One month ago, she was diagnosed with breast cancer. She refuses to listen tot he doctors recommendations and days, "Modern medicine never works, it's a waste of time! Herbs are always the best way!" Which of the following defense mechanisms is she most likely using? A. Projection B. Regression C. Splitting D. Denial E. Displacement F. Reaction formation

C. Splitting

A 25-year-old man comes to the physician because of a 3-month history of episodes of rage. His latest episode occurred after he put money into a vending machine at work. He knew that the vending machine was unreliable and randomly dispensed food after putting money into it. Despite the unreliability, he put money into the machine, and it did not dispense the purchased item. He became enraged and kicked the machine so hard that the glass on the machine broke. Which of the following schedules of reinforcement most likely accounts for why he put money into the machine despite its unreliability? A. Continuous B. Fixed ratio C. Variable ratio D. Fixed interval E. Variable interval

C. Variable ratio

A 35-year-old woman is to undergo a procedure in the nose under local anesthesia. For the operation to go as planned she needs an operating field that is bloodless and without any feeling. This can be mainly achieved by placing an anesthetic and vasoconstrictive agent near which of the following locations: A. Kiesselbach area B. infraorbital foramen C. sphenopalatine foramen D. anterior ethmoidal foramen E. posterior ethmoidal foramen

C. sphenopalatine foramen

A 30-year-old woman comes to the physician because of a 3-month history of numbness and tingling of the right half of her tongue. Neurological examination shows normal sensation of the lower lip and cheek on the right. Taste sensation is impaired on the anterior 2/3 of her tongue. Imaging studies show a lipoma in her right infratemporal fossa. Which of the following nerves is most likely injured/compressed by the lipoma? A.Inferior alveolar B.Chorda tympani C.Lingual nerve after junction with chorda tympani D.Buccal E.Lingual nerve before junction with chorda tympani F.Auriculotemporal

C.Lingual nerve after junction with chorda tympani -Chorda tympani is special sense, lingual nerve is general sense

The occipital part of the scalp is innervated by?

C2 and C3, and posterior auricular branch of facial

Discuss the clinical features of the 1st arch syndrome - Pierre Robin. What is the defect in this syndrome?

Causes micrognaathia/retrognathia, glossoptosis, and cleft palate.

A fractured left parietal bone would most likely cause what kind of hematoma?

Cephalohematoma (can see in delivery of a newborn)

What advantages are there in having the circumventricular organs with a weak BBB?

Check electrolyte balance, better detection in blood composition, and detects osmoregulation to change homeostasis so if you have high salt content, your hypothalamus can "manipulate" you into drinking more water; weak BBB helps you deliver hormones and nutrients more to their destination

Define counter conditioning and reciprocal inhibition and the role each plays in systematic desensitization.

Counterconditioning: presenting patient with anxiety inducing CS while relaxing so you replace unwanted response with new, wanted stimulus. Reciprocal inhibition is when anxiety and relaxation are mutually antagonistic and patient cannot experience them concurrently

25 y/o male comes to the physician because of a 3 year hx of alcohol use disorder. One component of his treatment involves aversion therapy. Which of the following stimuli is most likely to be paired with the CS associated with alcohol use? A. A pleasant conditioned stimuli B. An unpleasant conditioned stimuli C. A pleasant unconditioned stimulus D. An unpleasant unconditioned stimulus

D. An unpleasant unconditioned stimulus

A 1-week old girl is brought to the physician for a follow-up examination. She has a swelling on her head that was present after birth but is beginning to subside. Physical examination shows the swelling is subcutaneous, compressible and situated behind the vertex. Which of the following is the most likely diagnosis? A. Epidural hematoma B. Subdural hematoma C. Cephalhematoma D. Caput succedaneum E. Subgaleal hematoma

D. Caput succedaneum

A 22-year-old man comes to the physician because of a 1-year history of posttraumatic stress disorder. His symptoms started 3 months after being released from a prison of war (POW) camp. Part of his symptomatology includes experiencing fear whenever he encounters anything that reminds him of his POW experience. He now persistently avoids such reminders to reduce his fear. Which of the following learning principles best explains why he experiences fear whenever he encounters anything that reminds him of his POW experience? A. Spontaneous recovery B. Operant conditioning C. Operant extinction D. Classical conditioning E. Classical extinction

D. Classical conditioning

A 47-year-old man is brought to the emergency department by his partner because of a 5-hour history of severe vomiting and a total loss of vision in his right eye. Imaging studies show a lesion close to the right superior orbital fissure involving CN V1 and CN II. Physical examination shows a complete loss of vision in the right eye and numbness over the right forehead. Which of the following additional symptoms will most likely be found on further examination? A. Loss of lacrimation on the right side B. A dilated right pupil C. A constricted right pupil D. Diminished sensation of the tip of the nose E. Loss of sensation of the lower eyelid

D. Diminished sensation of the tip of the nose

A 22-year-old man comes to the physician because of a 3-year history of methamphetamine use. He has tried to stop using drugs on several occasions, but he relapses when he is around people, places, or things that have been associated with his drug use in the past. These stimuli from his drug-using environment make him crave the drug. Which of the following best describes a treatment plan for this patient based on classical extinction principles? A. Give him a monetary voucher each week that he remains abstinent from methamphetamine B. Fine him each time that he uses methamphetamine C. Expose him to methamphetamine and shock him if he uses the drug D. Expose him repeatedly to stimuli from his drug-using environment, but do not let him use methamphetamine E. Pair methamphetamine use with novel environmental stimuli

D. Expose him repeatedly to stimuli from his drug-using environment, but do not let him use methamphetamine

A 10-day old neonate comes to the hospital because of a 6-hour history of irritability and difficulty feeding. Physical examination shows a left-sided defect in her upper lip that continues into the hard palate up to the incisor foramen. Which of the following embryologic mechanisms will most likely explain the patient's condition? A. Failure of fusion of the intermaxillary segment with the maxillary prominences. B. Failure of fusion of the medial nasal prominences. C. Failure of fusion of the primary palate with the left lateral palatine processes D. Failure of fusion of the left maxillary prominence with the intermaxillary segment E. Failure of fusion of the left maxillary prominence to fuse with the lateral nasal prominence

D. Failure of fusion of the left maxillary prominence with the intermaxillary segment

A newborn baby girl is referred to the maxilla facial surgical department because of a left side defect in her upper lip. Oral exa: shows that the defect continues into the hard palate up to the incisor foramen. Which of the following embryonic mechanisms most likely explains the girls condition? A. Failure of fusion of the intermaxillary segment and the maxillary prominence B. Failure of fusion of the medial nasal prominences C. Failure of fusion of the primary palate to the lateral palatine process D. Failure of fusion of the maxillary prominence with the intermaxillary segment and of the primary palate to the lateral palatine process E. Failure of the fusion of the lateral palatine processes with the primary palate and with each other

D. Failure of fusion of the maxillary prominence with the intermaxillary segment and of the primary palate to the lateral palatine process

1. Palliative care means that the patient has forfeited curative treatment2. Palliative care means that the patient is terminally ill with ≤6 months to live A. True; True B. True; False C. False, True D. False, False E. No idea

D. False, False

A 21-year-old woman comes to the physician because of a 10-year history of a fear of enclosed spaces. She dates the origin of her fear to a childhood experience in which she was trapped in a small elevator in a shopping mall for several hours when it malfunctioned. Due to her fear of enclosed spaces, she fears and avoids getting into an MRI scanner, which is needed to help diagnose the cause of her lower back pain. Which of the following is most likely serving as the conditioned response? A. Elevators B. Being trapped in a small malfunctioning elevator C. Lower back pain D. Fear and avoidance E. Enclosed spaces (like an MRI scanner)

D. Fear and avoidance

A 12-year-old boy is brought to the physician by his mother for a follow-up examination. For the past 6 months he has received treatment for attention-deficit/hyperactivity disorder. His attention at school has improved in part because of a behavioral intervention implemented by his teacher. The boy can earn tokens for working attentively, and these tokens can be traded at the end of the day for a desirable item such as food or stickers. The teacher dispenses the tokens after every 30 minutes of class time. Which of the following operant schedules best describes the method being used to reinforce his attentive behavior in class? A. Continuous B. Fixed ratio C. Variable ratio D. Fixed interval E. Variable interval

D. Fixed interval

After a 6 month long semester of medical school in Grenada, a 26 year old medical student comes tot he US and ord3s a Big Mac. 10 minutes later, he cannot close his jaw to chew on his food. He was taken to the ED and diagnosed with anterior dislocation of the jaw. Which of hotel following muscles is responsible for depression of the mandible? A. Massager B. Temporalis C. Medial pterygoid D. Lateral pterygoid

D. Lateral pterygoid

A 21-year-old professional boxer is brought to the emergency department by his colleague because of a severe pain in his left jaw due to a punches during boxing game. Temperature 37.2 C , pulse is 85/min, blood pressure is 126/79 mm Hg, respiration is 19/min. Physical examination shows teeth loss, numbness over the jaw and trismus (difficulty opening the mouth). Imaging is showed below. Which of the following muscles attached to the outer aspect of the fractured area? A. Temporalis B. Lateral pterygoid C. Zygomaticus major D. Masseter E. Posterior digastric

D. Masseter -think muscles of mastication

An 8-month-old infant is brought to the physician by her mother for a routine examination. Physical examination shows an infant who does not turn and open her mouth when her cheek is stroked. When the sole of her foot is stroked, her toes curl upwards and spread. When her palm is stroked, she does not clench her fist. While supporting her head, back and legs, the physician abruptly lowers her entire body. Her arms abducted and her legs flexed. Which of the following primitive newborn reflexes are abnormal in this infant? A. Rooting reflex B. Babinski reflex C. Palmar grasp reflex D. Moro reflex E. Sucking reflex

D. Moro reflex -8-month-old infant -does not turn and open her mouth when her cheek is stroked (normal for her age) -sole of her foot is stroked, her toes curl upwards and spread (normal for her age) -palm is stroked, she does not clench her fist (normal for her age) -While supporting her head, back and legs, the physician abruptly lowers her entire body. Her arms abducted and her legs flexed (abnormal)

A 6-month-old infant comes to the hospital because of a 12-hour history of irritability, poor feeding and projectile vomiting. Physical examination shows a bulging forehead, uneven cheekbones and an undeveloped left eye socket. CT scan reveals bony abnormalities seen in the image provided. Which of the following most likely explains these finding? A. Trigonocephaly B. Scaphocephaly C. Brachycephaly D. Plagiocephaly E. Cranium bifidum

D. Plagiocephaly

A 6-month-old infant comes to the hospital because of a 12-hour history of irritability, poor feeding and projectile vomiting. Physical examination shows a bulging forehead, uneven cheekbones and an undeveloped left eye socket. CT scan reveals the bony abnormalities seen in the image provided. Which of the following most likely occurred to explain the patient's findings? A. Early closure of the frontal sutures. B. Congenital stenosis of the apertures between the ventricles. C. Early closure of the sagittal suture D. Premature closure of the coronal suture E. Failure of fusion of the cranial neuropore

D. Premature closure of the coronal suture

A 55-year-old woman is brought to the hospital by her husband because of a 5-month history of progressively increasing pain due to bone cancer. She is receiving chemotherapy as treatment. The physician mentions the possibility of palliative care, but she is resistant. Which of the following facts is most likely to encourage her to pursue palliative care? A. Needing to give up taking chemotherapy B. Receiving her care at home rather than in the hospital C. Being eligible for additional specific invasive treatments for her cancer D. Receiving additional support for coping with her disease E. Receiving enhanced assessment and management of medical complications

D. Receiving additional support for coping with her disease

A 45-year-old women comes to the physician because of difficulty in swallowing, rapid breathing and change of voice (monotonous voice tone) for the last 2 weeks. She have a family history of multiple endocrine neoplasia type 2 (MEN II). Temperature 37.8 C (100.1F), pulse is 78/min, blood pressure is 145/88 mm Hg, respiration is 24/min. Physical examination shows a solitary thyroid nodule, 3x5 inches, extending retrosternally. Fine-needle aspiration (FNA) biopsy confirmed the diagnosis of papillary cell carcinoma which was removed surgically.Post-surgical examination shows unilateral paralysis of the vocal cords. Which of the following nerves is affected? A. Superior laryngeal nerve B. External laryngeal nerve C. Internal laryngeal nerve D. Recurrent laryngeal nerve E. Pharyngeal branch of glossopharyngeal F. Pharyngeal branch of the vagus

D. Recurrent laryngeal nerve

The external laryngeal n. of a 2e year old man becomes ensnared and tightly compressed by a tortuous superior thyroid a., which parallels the course of this nerve. Which of the following functions is most likely to be affected? A. Sensation above the true vocal fold B. Sensation below the true vocal fold C. Abduction of the vocal cord D. Tension of the vocal cord E. Depression of the hyoid bone

D. Tension of the vocal cord

Activation of chemoreceptors in the area postrema would cause which of the following? A. Headache B. Nuchal rigidity C. Photophobia D. Vomiting E. Papilledema

D. Vomiting

A 63-year-old amateur singer comes to the clinic because of a husky voice for 3 weeks. Examination of the larynx shows that the right vocal cord is paralyzed and does not meet the midline on phonation. There is some pooling of saliva in the right piriform sinus as well. The physician is thinking of which of the following possibilities as the most likely underlying cause? A. Aneurysm of the subclavian artery B. Lesion of the pharyngeal branch of CNX C. Lesion of superior laryngeal nerve D. Upper esophageal cancer E. Metastasis of lung cancer around the hilum of the lung

D. upper esophageal cancer (yes, space occupying lesion and tracheo-esophageal groove where the right recurrent laryngeal nerve travels) A. aneurysm of the subclavian artery (that would affect the right recurrent laryngeal nerve but not the pooling of saliva) B. lesion of the pharyngeal branch of CNX (does not innervate the vocal fold, becomes part of pharyngeal plexus and is sensory) C. lesion of superior laryngeal nerve (does not innervate the vocal fold) E. metastasis of lung cancer around the hilum of the lung (that could be if it was the left side but pooling on the right does not fit)

A 50-year-old man comes to the physician because of a 3-day history of pain below the right jaw shortly after he eats. On physical examination a small hard swelling is felt below the tongue and imaging shows a calculus in the right submandibular duct. During surgery for removal of the stone trauma to the nerve that wraps around the submandibular duct occurred. Which of the following symptoms will most likely be present in this patient as a result of the nerve being severed? A.Weakened ability to protrude the tongue B.Weakened ability to depress the mandible C.Weakened ability to chew D.Decreased general sensation from the anterior 2/3rds of the tongue on the affected side E.Decreased general sensation from the posterior 1/3rd of the tongue on the affected side

D.Decreased general sensation from the anterior 2/3rds of the tongue on the affected side

A 20-year-old man comes to the emergency department because of a 1-day history of headache, nausea and a painful scalp. He received a small laceration to the anterior aspect of his scalp 2 days ago, which he has been self-treating. Physical examination shows the laceration is infected and has spread posteriorly along the scalp. Through which of the following layers of the scalp has this infection most likely spread? A.Skin B.Dense connective tissue C.Aponeurosis D.Loose connective tissue E.Periosteum

D.Loose connective tissue

A 50-year-old man comes to the emergency department because of a 1-week history of an ulcer on his lower lip. He has a 20 pack year history of smoking. Which of the following lymph nodes would most likely be enlarged on palpation? A.Submandibular B.Occipital C.Preauricular D.Submental E.Superior deep cervical F.Supraclavicular

D.Submental

A 72-year-old woman is brought to the emergency department because of a 2-hour history of chest pains. She says the pain feels like a crushing sensation that is moving to her left shoulder and jaw. She has a 10-year history of uncontrolled hypertension. Her blood pressure is 150/90 mmHg, pulse is 120/min and respirations are 14/min. She becomes very upset and refuses to be examined by the physician. Which of the following is the most appropriate response from the physician? A. Walk away B. "I cannot help you if you do not let me examine you". C. "Let me know when you would like me to help you." D. "I understand you are in pain but you need to let me examine you so we can make a diagnosis". E. "May I ask what is upsetting you so that I can help you as best I can?"

E. "May I ask what is upsetting you so that I can help you as best I can?"

***A 1-day-old boy is admitted to the pediatric care unit because of problems experienced when trying to suck the breast. Physical examination shows a left posterior cleft palate. Which of the following best describes the embryological development of the secondary palate? A. Develops from the second pharyngeal arch B. Is formed by the median palatine process C. Formed by fusion of the medial nasal processes D. Begins as two vertical outgrowths from the developing mandible E. Begins as two lateral outgrowths from the developing maxilla

E. Begins as two lateral outgrowths from the developing maxilla

A 40-year-old man comes to the physician because of a 1-year history of depression. He says that none of his relationships last and "no one will ever love me." The physician identifies, challenges, and corrects the patient's irrational way of thinking. Which of the following therapies is the physician most likely using? A. Cognitive-Behavioral B. Aversion C. Flooding D. Exposure E. Cognitive

E. Cognitive

A 35-year-old woman comes to the physician for a follow-up examination of her terminal cancer. Her spirits have been lifted because she has been feeling better the past week. She believes that her improvement is a sign that her prayers have been answered. Which of the following stages of grief is she most likely to experience next? A. Anger B. Acceptance C. Bargaining D. Denial E. Depression F. There is no next stage

E. Depression

A 63-year-old man is brought by his wife to the emergency department because of difficulty opening his right eye for the last 4 hours. He has a 15 year history of diabetes. Temperature 37.8 C (100.1F), pulse is 68/min, blood pressure is 132/83 mm Hg, respiration is 17/min. Physical examination shows right upper lid ptosis. CT scan shows no signs of subarachnoid hemorrhage. Which of the following signs should be identified during physical examination? A. Diplopia when the patients looks to the right B. Loss of consensual eye light reflex on the left C. Loss of the light reflex of the left eye D. loss of tears in the right eye E. Dilation of right pupil

E. Dilation of right pupil -unable to open eye = full ptosis (oculomotor nerve = parasympathetic = constrict) or partial ptosis (sympathetic = dilation of pupil) -this one is more acute

A 35-year-old woman is brought to the emergency department by her husband with a 1-week history of strange behavior. She believes she was chosen to solve a mystery that will save all of humankind, and so spends her days looking for clues. She has been interrogating everyone she meets but they are often distracted by the sing-song melody of her speech. Which of the following is most likely the best term to describe her speech? A. Echolaic B. Dysfluent C. Hypophonic D. Stilted E. Dysprosodic F. Pressured

E. Dysprosodic

A 58 y/o woman comes to the physician because of a 6 month hx of depression. She says "I am tired of feeling this way. I just want to feel better". She says that she has lost all desire to d o things that used to be fun. Some days she feels so sad she cannot get out of bed in the morning, and she has lost all desire to eat and sleep. She describes. Sadness as pervasive and chronic. Which of the following aspects the "HPI" is most likely missing from this patients intake interview? A. Symptom onset B. Symptom duration C. Symptom intensity D. Symptom quality E. Environmental context F. Why not?

E. Environmental context

A 10 year old boy is brought to the ED by his father because of sore throat, ear ache and high fever. Temp (38.8 degrees Celsius), BO (135/84 mmHg), and respiration 24/min. PE shows red and swollen palatine tonsils (tonsillitis). What nerve carries the sensory input for most of the patients symptoms? A. Greater palatine B. Lesser palatine C. Vagus D. Posterior superior alveolar E. Glossopharyngeal

E. Glossopharyngeal

A 5 year old boy is brought to the physician by his parents for a routine exam. He is developing normally across most domains, except for one area of functioning . Which of the following findings is most likely to be unusual for his age and evoke the most concern?

E. He lacks cooperative play

A 45-year-old man comes to the physician for a routine examination. He has always been a loner who does not like to be around other people. He has never been in a significant relationship. He has a stable sense of self. Which of the following stages of psychosocial development was most likely influential in the formation of his dominant personality trait? A. Trust vs mistrust B. Industry vs inferiority C. Identity vs role confusion D. Autonomy vs shame and doubt E. Intimacy vs isolation

E. Intimacy vs isolation

A 65-year-old man is brought to the emergency department because of a 2-hour history of shortness of breath. He is coughing and wheezing. He has a 10-year history of chronic obstructive pulmonary disease. Physical examination shows a restless man with labored and shallow breathing. His respirations are 24/min. The physician suspects an acute exacerbation of his chronic obstructive pulmonary disease. He is admitted to ICU for further management. Which of the following pieces of information is considered part of the objective section of a SOAP note? A. Shortness of breath B. History of chronic obstructive pulmonary disease C. Acute exacerbation of chronic obstructive pulmonary disease D. Admission to ICU E. Labored and shallow breathing

E. Labored and shallow breathing

A 26-year-old man comes to the physician because of a 2-year history of hearing problems. Weber test shows lateralization to the left. Which of the following patterns of hearing loss best explains the results of the test? A. Right conductive B. Left sensorineural C. Left conductive or left sensorineural D. Right conductive or right sensorineural E. Left conductive or right sensorineural F. Right conductive or left sensorineural

E. Left conductive or right sensorineural

A 21-year-old man comes to the physician because of a shooting pain in his lower jaw. He complains that it is the most severe pain he has ever felt but there is no muscle weakness. A diagnosis of mandibular trigeminal neuralgia is made. Through which of the following foramina does the affected nerve exit the skull? A. Rotundum B. Superior orbital fissure C. Inferior orbital fissure D. Spinosum E. Ovale

E. Ovale

A 43-year-old woman is brought to the emergency department because of a 1-day history of agitation and pain. She has a broken leg. She closes her eyes against the bright lights but cannot drown out the noise around her. The telephone has been ringing nonstop for the past ten minutes. She opens her eyes and yells at a nurse, "Answer the tephelone!" Based on this statement, which of the following terms most likely describes her thought process/content? A. Ruminative B. Neologistic C. Paragrammatic D. Clanging E. Paraphasia

E. Paraphasia

A 55-year-old man comes to the physician because of a 3-day history of dimming of vision. Imaging studies shows an infarction involving the left optic tract. Which of the following visual field defects is most likely? A. Right superior homonymous quadrantanopia B. Left superior homonymous quadrantanopia C. Right inferior homonymous quadrantanopia D. Left inferior homonymous quadrantanopia E. Right homonymous hemianopia F. Left homonymous hemianopia G. Superior homonymous hemianopia H. Inferior homonymous hemianopia

E. Right homonymous hemianopia

A 65-year-old man comes to the emergency department because of persistent nosebleed for the last 4 hours. Temperature 37.8 C (100.1F), pulse is 78/min, blood pressure is 145/88 mm Hg, respiration is 24/min. Physical examinations shows nail clubbing, palmer erythema and spider nevi. Examination of the nose shows active bleeding in the anterior aspect of the of the nasal cavity. The bleeding is controlled by direct pressure. Which of the following arteries contribute to the bleeding area? A. Anterior superior alveolar arteries B. Infraorbital C. Facial D. Posterior ethmoidal E. Sphenopalatine

E. Sphenopalatine and D. Posterior ethmoidal KNOW THIS!!!!

A 67-year-old man comes to the physician because of a 6-week history of depression. He just had a suicide attempt. He said that he misses daily conversations and interactions with his wife who passed away 6 weeks ago. After her death, his children moved back home to be with him. He says that he has lived a full life, and he knows that his children will be just fine without him. He started going back to the church where he and his wife used to go, and says that is what she would want him to do. Which of the following factors is most likely a protective factor against future suicide attempts? A. Failed suicide attempt B. Sense of responsibility to family C. Life satisfaction D. Wife's death E. Spirituality

E. Spirituality

A 24-year-old woman comes to the physician because of a 4-day history of a progressively worsening painful swelling to her right eyelid. Physical examination shows a pus-filled reddish lump on the outer edge of her right upper eyelid as shown in the attached image. Biopsy shows that there is obstruction of the sebaceous glands associated with the eyelashes. Which of the following glands is most likely affected? A. Wolfring B. Krause C. Meibomian D. Moll E. Zeis

E. Zeis

A swelling in the neck that moves up and down during swallowing is most likely originating from which of the following triangles: A. posterior B. anterior C. submandibular D. carotid E. muscular

E. muscular

A 67-year-old woman comes to the physician for a follow-up examination six weeks after suffering a stroke. Neurologic examination shows that her uvula deviates to the left when she is asked to say 'ahhhh'. Imaging studies show a tumor compressing a cranial nerve. Which of following nerves most likely innervates the paralyzed structures? A.Left glossopharyngeal B.Left vagus C.Right glossopharyngeal D.Right spinal accessory E.Right vagus

E.Right vagus

A 55 y/o man is brought to the physician by his wife because of a. 1 week hx of erratic behaviors. The wife says that he has been going on shopping speed and spending hours online gambling. She also says that he has felt depressed for the past several months. He has been taking dopamine agonists for the tx of Parkinson's disease for the past 12 months. During the interview, the man initially appears cheerful and talkative but eventually becomes despondent, slumped, and unresponsive. Which of the following terms best describe his mood and affect? A. Anxious, blunted B. Depressed, flat C. Elated, euphoric D. Irritable, depressed E. Anxious, euphoric F. Depressed, labile G. Elated, depressed

F. Depressed, labile

A 62-year-old man is brought to the emergency department because he was found unconscious in his house. His neighbor says that he called the police because he suspected that something was wrong because he did not see him for 2 days. A CT scan of his head shows several lesions. Which of the following symptoms is the patient likely to have when he regains consciousness? A. Right hemianopia, right hemiparesis, right mydriasis B. Right hemianopia, left hemiparesis, right mydriasis C. Right hemianopia, right hemiparesis, left mydriasis D. Right hemianopia, left hemiparesis, left mydriasis E. Left hemianopia, right hemiparesis, right mydriasis F. Left hemianopia, left hemiparesis, right mydriasis G. Left hemianopia, right hemiparesis, left mydriasis H. Left hemianopia, left hemiparesis, left mydriasis

F. Left hemianopia, left hemiparesis, right mydriasis

A 21-year-old man is brought by the ambulance to the emergency department after a motor vehicle accident 1 hour ago. Temperature 37.8 C (100.1F), pulse is 55/min, blood pressure is 155/97 mm Hg, respiration is 17/min. Physical examination shows decrease of the level of consciousness and bruise on the right side of the head. He extends his left arm and leg to painful stimuli, but there is no movement on the right side. X- ray shows fracture on the skull 2 cm above and anterior to the right pinna. Which of the following blood vessels is affected in the patient? A. Anterior communicating artery B. Posterior communicating artery C. Posterior cerebral artery D. Anterior cerebral artery E. Middle cerebral artery F. Middle meningeal artery G. Posterior meningeal artery H. Anterior meningeal

F. Middle meningeal artery

Which of the following neurotransmitters is chiefly released by postganglionic sympathetic neurons onto visceral targets? A. Dopamine B. Serotonin C. Acetylcholine D. GABA E. Glutamate F. Norepinephrine

F. Norepinephrine

A 33-year-old woman is brought to the emergency department by ambulance because of a 2-hour history of problems walking. Neurological examination shows paresis of her left upper and lower extremities. Touch, vibration, proprioception, pain and temperature sensations are absent in the left side of the body and face. Which of the following structures is most likely damaged? A. Left spinal cord B. Right spinal cord C. Left thalamus (VPL and VPM) D. Right thalamus (VPL and VPM) E. Left internal capsule F. Right internal capsule

F. Right internal capsule

Two of the boundaries of the posterior triangle are innervated by the same nerve which is entering the skull through which foramen?

Foramen magnum

Which paranasal sinuses can be found at the orbit?

Frontal, ethmoid and maxillary sinuses

A 75-year-old man comes to the physician because of a 2-day history of problems with his vision. Imaging shows a left temporal lobe lesion. Which of the following visual field defects most likely affects this patient? A. A (r. Eye black) B. B (left superior bilaterally) C. C (left side bilaterally) D. D (left inferior bilaterally) E. E ( left superior bilaterally without the center) F. F (left eye black) G. G (right superior bilaterally) H. H (right side bilaterally) I. I (right inferior bilaterally) J. J (right superior bilaterally without the center)

G. G (right superior bilaterally)

Would massaging the carotid sinus have any consequences in the above case

Good to diagnose tachyarrhythmias, so it's won't cause any changes to what's currently happening

The area of skin over angle of the mandible is said to be innervated by?

Great auricular nerve, which is part of the cervical branch

A 70-year-old woman comes to the physician for a routine examination. She overall feels healthy but has noticed a slight decline in her thinking abilities over the past 12 months. The physician performs a mental status examination to briefly assess her cognitive skills. On which of the following cognitive tasks is she most likely to show normal age-related decline? A. Reading words B. Defining vocabulary words C. Comprehending written language D. Comprehending spoken language E. Knowing facts about the world F. Naming objects G. Repeating phases H. Processing new information at a rapid pace

H. Processing new information at a rapid pace

Which interactions between HPA, HPG & HPT would you expect?

HPA - Cortisol/ACTH with glucocorticoids HPG- Gonads (LH/FSH) HPT- Thyroid (TSH) Inc. ACTH = Inc. cortisol = dec. GnRH = inc. TSH and TRH

Which bone in the body does not articulate with another bone but moves freely?

Hyoid bone

****A 49-year-old woman comes to the physician because of 24-hour history of problems smelling. Neurological examination shows a complete loss of olfactory function bilaterally. MRI shows a local edema and bleeding. Which of the following structures is most likely lesioned bilaterally? A. Precentral gyrus B. Postcentral gyrus C. Anterior paracentral gyrus D. Posterior paracentral gyrus E. Transverse temporal gyrus F. Inferior frontal gyrus - pars opercularis G. Inferior frontal gyrus - pars triangularis H. Inferior frontal gyrus - pars orbitalis I. Gyrus rectus

I. Gyrus rectus -path to the uncus;

Which structures are situated medically to the hyoglossus muscle and which are laterally located?

Medial: tongue muscles, CN 9 and lingual artery (branch of ECA) Lateral: submandibular gland and duct, CN 12

Discuss the clinical features of the 1st arch syndrome - Treacher Colins. What is the defect in this syndrome?

Micrognathia, 50% cleft palate and lip, conductive hearing loss, undrdeveloped zygoma, malformed pinna, and drooping of lateral lower eyelids -ALL D/T failure of migration of neural crest cells into 1st arch (Know 1st arch derivative to know symptoms/signs)

What part of the clavicle is most susceptible to fracture?

Middle 3rd clavicle (distal and from the lateral parts); most susceptible because there is no muscle or ligamentous supporting it and that's where you have the thinnest pad of the muscle. The arm is distal to it so there's more weight with rotation of the shoulder

Would you expect the weighting attached to the specific stressors in the previous slide to be the same for different people?

No, because people respond to stress differently. Some can cope and tolerate the stressors, and for others it can be too much for them to handle.

Which muscle makes the hyoid bone move? What is the innervation of each of hose muscles?

Picks up genohyloid (CN 1), and mylohyoid ( CN V3), omohyoid (ansa cervicalis) and digastric with stylohyoid (facial n.)

In which of triangle would that fracture be located?

Posterior triangle

Given the functions of the anterior pituitary, what would the effects of a lesion to this structure?

Prolactinoma (DA), low sex drive, decreased growth regulation (GH), decreased lactogenesis, high TRH, low TSH and decreased T3 and T4 (impaired metabolic regulation and hypothyroidism), impaired stress response (ACTH),

Identify components of the acute stress response which can lead to negative health consequences in case of chronic stress

Someone with CAD would have completely occluded arteries as a result of additional stress, leading to a possible cardiac arrest Chronic inc. cortisol = atherosclerosis, inflammation, high blood glucose and high FFA, HTN, dec. immunity = dec. interleukin = lymphopenia and neutropenia, heightened auto-immune responses (can inc. MS, or juvenile diabetes)

Lymphatic drainage of the tongue is important to know because of cancerous lesion of the tongue, also the sensory innervation is important to know because of referred pain from cancerous tongue lesions. A tip of the tongue lesion wipould drain to which lymph nodes and could cause referred pain where? A lesion of the vallecula would most likely drain to which lymph nodes and cause what kind of referred pain?

Sub mental and referred pain tot he ear (ear ache) because they're both innervated by the lingual nerve, which is sensory supply to the tongue; sub mental and pain from Glossopharyngeal nerve

In which later of tissue is the platysma found?

Subcutaneous fascia (adipose tissue)

A person with long hair gets his ht in machinery. It takes a while to find the switch to turn off the machine, causing what kind of bleed?

Subgaleal hemorrhage

What could be the clinical importance of that frontal sinus being at the orbit?

Subject to inflammation that may cause pain over the affected sinus (e.g. frontal headache due to frontal sinusitis) and they can end up with exothalmus or eye infection; also consider cancer; during surgery, if you remove the nasal polyp you can cause issues a the ethmoid sinus

What is the difference between the temporality nerve and the temporal nerve?

Temporalis = CN 8 Temporal = CN 7

Discuss the embryological development of the thyroid gland. What are some of the abnormalities that can occur?

Thyroid is an endodermal thickening in floor of pharynx that descends down and fuses with the tongue at foramen cecum to create the thyroglobulin duct, which degenerates and disappears normally. Anomalies: ectopic thyroid - abnormal position of thyroid Pyramidal lobe - persistent part of thyroglossal duct Thyroglossal duct cyst and sinus (how is the cyst different from goiter?)

The frontal part of the scalp is innervated by?

Trigeminal n. and temporal branch of facial n.

How does a patient present with a lesion of the facial nerve at the stylomastoid foramen?

Unable to smile, facial droop especially at the mouth, can't raise eyebrow (eyebrow raises if it's an UMN lesion because UPPER SPARES UPPER because there's bilateral supply from the cortex, but if it's a LMN then whole face is in trouble because there is only one supply going to the LMN)

What roots form the spinal accessory nerve?

Ventral rami (C1-C5), then ascends up to the foramen magnum

A 63 year old man enjoys his meal in a steal house until he suddenly becomes very quiet, has difficulty breathing and begins to turn blue in the face. A Heimlich maneuver by one of the waiters of the establishment has not restored the breathing. What are you now contemplating and explain.

You apply pressure on the lower part of the lounge so the air there pushes out whatever is constructing your airway. If that doesn't work, do a: -A tracheostomy is a surgical opening to access the tracheal lumen with the entire larynx remaining intact (D). In contrast, after total laryngectomy, the trachea is brought to the skin as a stoma, which no longer has any anatomical connection with the oropharyngeal cavity and digestive tract (C). The Adam's apple is the thyroid cartilage and you cut between the thyroid cartilage and cricoid cartilage

Where does the vagus n. travel in the neck? Where is the origin of the vagus n.? Think of the consequences if the nerve is severed in the jugular foramen.

a. Between common carotid a. and IJV b. CN 10, in the lateral medulla (post-olivary sulcus, with CN 9 and 11) c. Troubled speech because it supplies larynx and pharynx, also would affect the intestine too (vomiting, cannot digest food)

A 37-year-old man comes to the physician because of severe pain over his upper front teeth on the right. He complains that he has been sneezing constantly over the past 3 weeks and that his nose has been blocked for the past week. Physical examination shows puffy eyes with pale mucosa, swollen pale colored nasal mucosa and pain on palpation just below the right orbit. The nerve responsible for the pain over his teeth passes through which of the following foramina? a. Infraorbital b. Inferior orbital fissure c. Pterygomaxillary fissure d. Foramen ovale e. Pterygoid canal

a. Infraorbital

Where does the phrenic nerve travel, which spinal nerves contribute to this nerve, what is the consequence of a left side design , what is the consequence of a bilateral lesion?

a. On top of anterior scalene m. b. C3-5 c. Left sided lesion: affect diaphragm and respiratory problems of left side d. Bilateral: respiratory failure

Identify where a laryngotomy and a tracheotomy would be done. Which nerve should be anesthetize for such procedures? Indicate the level of the true vocal cords.

b. Cervical nerves and for a tract, include CN 10 c. Laryngeal prominence/Adams apple (1cm above cricoid cartilage)

A 53-year-old man is diagnosed with a tumor at the base of the skull, resulting in a decrease in tear secretion. Which of the following nerves is most likely injured in this patient? a. Chorda tympani b. Greater petrosal c. Lesser petrosal d. Nasociliary e. Deep petrosal

b. Greater petrosal (and zygomatic, but it's the main nerve that innervates the lacrimal gland)

A 36-year-old man is brought to the emergency department with severe head injuries after a car crash. During neurological examination it was noted that his uvula is deviated to the right. Which of the following nerves is most likely affected in this patient? a. Left glossopharyngeal b. Right glossopharyngeal c. Left vagus d. Right vagus e. Left hypoglossal f. Right hypoglossal

c. Left vagus

A 65-year-old man comes to the physician because of nasal congestion for the past two weeks. He complains of pain over his cheeks and upper teeth as well as continual mucous discharge from his nose which has now turned green. Physical examination shows red swollen nasal mucosa and closed nasal passage. The nerve most likely responsible for the pain over his cheek and upper teeth is best described to take which of the following routes? a. Middle cranial fossa —> foramen ovale —> IT fossa —> mandibular foramen b. Posterior cranial fossa —> foramen ovale —> IT fossa —> inferior orbital fissure —> infraorbital foramen c. Middle cranial fossa —> foramen rotundum —> PT fossa —> inferior orbital fissure —> infraorbital foramen d. Posterior cranial fossa —> foramen rotundum —> PT fossa —> inferior orbital fissure —> infraorbital foramen e. Middle cranial fossa —> foramen ovale —> PT fossa —> superior orbital fissure —> infraorbital foramen

c. Middle cranial fossa —> foramen rotundum —> PT fossa —> inferior orbital fissure —> infraorbital foramen

The muscles of mastication are innervated by the mandibular nerve. Which of the following muscles is also innervated by a branch of the mandibular nerve? a. Stapedius b. Posterior belly of digastric c. Buccinator d. Tensor pallatini muscle

d. Tensor pallatini muscle

A 61-year-old man is admitted to the emergency department with severe hypertension. His bp is 190/110 mm Hg. Laboratory investigations reveals hypercholesterolemia. Radiological examination reveals 90% occlusion of both the common carotid arteries. A carotid endarterectomy is performed, and atherosclerotic plaques are removed. It was noted during the post-operative examination that his tongue deviated to the left when asked to stick it out. Which of the following nerves is most likely injured in this patient? a. Left glossopharyngeal b. Right glossopharyngeal c. Left vagus d. Right vagus e. Left hypoglossal f. Right hypoglossal

e. Left hypoglossal

A 31-year-old woman is brought to the emergency department with severe headaches for the past 5-days. Imaging studies reveals a tumor in the infratemporal fossa. Physical examination reveals loss of general sensation from the anterior two-thirds of his tongue, but special sensation is intact. Which of the following nerves is most likely affected in this patient? a. Chorda tympani b. Inferior alveolar c. Lessor petrosal d. Glossopharyngeal e. Lingual

e. Lingual -taste (special) sensation: chordai tympani -general sensation : lingual


Ensembles d'études connexes

Unit 6: Claims and Evidence - Reading Quiz

View Set

History 800B Lesson 6 - Events Leading To Succession And War

View Set

Early US History Chapter 10 Study Guide

View Set

BIOL 3500: Case study (T cell: B cell interaction) 3/9

View Set